Skip to main content
Mathematics LibreTexts

6.4E: Exercises

  • Page ID
    35383
  • \( \newcommand{\vecs}[1]{\overset { \scriptstyle \rightharpoonup} {\mathbf{#1}} } \) \( \newcommand{\vecd}[1]{\overset{-\!-\!\rightharpoonup}{\vphantom{a}\smash {#1}}} \)\(\newcommand{\id}{\mathrm{id}}\) \( \newcommand{\Span}{\mathrm{span}}\) \( \newcommand{\kernel}{\mathrm{null}\,}\) \( \newcommand{\range}{\mathrm{range}\,}\) \( \newcommand{\RealPart}{\mathrm{Re}}\) \( \newcommand{\ImaginaryPart}{\mathrm{Im}}\) \( \newcommand{\Argument}{\mathrm{Arg}}\) \( \newcommand{\norm}[1]{\| #1 \|}\) \( \newcommand{\inner}[2]{\langle #1, #2 \rangle}\) \( \newcommand{\Span}{\mathrm{span}}\) \(\newcommand{\id}{\mathrm{id}}\) \( \newcommand{\Span}{\mathrm{span}}\) \( \newcommand{\kernel}{\mathrm{null}\,}\) \( \newcommand{\range}{\mathrm{range}\,}\) \( \newcommand{\RealPart}{\mathrm{Re}}\) \( \newcommand{\ImaginaryPart}{\mathrm{Im}}\) \( \newcommand{\Argument}{\mathrm{Arg}}\) \( \newcommand{\norm}[1]{\| #1 \|}\) \( \newcommand{\inner}[2]{\langle #1, #2 \rangle}\) \( \newcommand{\Span}{\mathrm{span}}\)\(\newcommand{\AA}{\unicode[.8,0]{x212B}}\)

    Exercise \(\PageIndex{1}\)

    For the following exercises, use the information provided to solve the problem.

    1) Let \(\displaystyle w(x,y,z)=xycosz,\) where \(\displaystyle x=t,y=t^2,\) and \(\displaystyle z=arcsint.\) Find \(\displaystyle \frac{dw}{dt}\).

    Answer

    \(\displaystyle \frac{dw}{dt}=ycosz+xcosz(2t)−\frac{xysinz}{\sqrt{1−t^2}}\)

    2) Let \(\displaystyle w(t,v)=e^{tv}\) where \(\displaystyle t=r+s\) and \(\displaystyle v=rs\). Find \(\displaystyle \frac{∂w}{∂r}\) and \(\displaystyle \frac{∂w}{∂s}\).

    3) If \(\displaystyle w=5x^2+2y^2,x=−3s+t,\) and \(\displaystyle y=s−4t,\) find \(\displaystyle \frac{∂w}{∂s}\) and \(\displaystyle \frac{∂w}{∂t}\).

    Answer

    \(\displaystyle \frac{∂w}{∂s}=−30x+4y, \frac{∂w}{∂t}=10x−16y\)

    4) If \(\displaystyle w=xy^2,x=5cos(2t),\) and \(\displaystyle y=5sin(2t)\), find \(\displaystyle \frac{∂w}{∂t}\).

    5) If \(\displaystyle f(x,y)=xy,x=rcosθ,\) and \(\displaystyle y=rsinθ\), find ∂f∂r and express the answer in terms of \(\displaystyle r\) and \(\displaystyle θ\).

    Answer

    \(\displaystyle \frac{∂f}{∂r}=rsin(2θ)\)

    6) Suppose \(\displaystyle f(x,y)=x+y,u=e^xsiny,x=t^2\) and \(\displaystyle y=πt\), where \(\displaystyle x=rcosθ\) and \(\displaystyle y=rsinθ\). Find \(\displaystyle \frac{∂f}{∂θ}\).

    Exercise \(\PageIndex{2}\)

    For the following exercises, find \(\displaystyle \frac{df}{dt}\) using the chain rule and direct substitution.

    7) \(\displaystyle f(x,y)=x^2+y^2, x=t,y=t^2\)

    Answer

    \(\displaystyle \frac{df}{dt}=2t+4t^3\)

    8) \(\displaystyle f(x,y)=\sqrt{x^2+y^2},y=t^2,x=t\)

    9) \(\displaystyle f(x,y)=xy,x=1−\sqrt{t},y=1+\sqrt{t}\)

    Answer

    \(\displaystyle \frac{df}{dt}=−1\)

    10) \(\displaystyle f(x,y)=\frac{x}{y},x=e^t,y=2e^t\)

    11) \(\displaystyle f(x,y)=ln(x+y), x=e^t,y=e^t\)

    Answer

    \(\displaystyle \frac{df}{dt}=1\)

    12) \(\displaystyle f(x,y)=x^4, x=t,y=t\)

    Exercise \(\PageIndex{3}\)

    13) Let \(\displaystyle w(x,y,z)=x^2+y^2+z^2, x=cost,y=sint,\) and \(\displaystyle z=e^t\). Express \(\displaystyle w\) as a function of \(\displaystyle t\) and find \(\displaystyle \frac{dw}{dt}\) directly. Then, find \(\displaystyle \frac{dw}{dt}\) using the chain rule.

    Answer

    \(\displaystyle \frac{dw}{dt}=2e^{2t}\) in both cases

    14) Let \(\displaystyle z=x^2y,\) where \(\displaystyle x=t^2\) and \(\displaystyle y=t^3\). Find \(\displaystyle \frac{dz}{dt}\).

    15) Let \(\displaystyle u=e^xsiny,\) where \(\displaystyle x=t^2\) and \(\displaystyle y=πt\). Find \(\displaystyle \frac{du}{dt}\) when \(\displaystyle x=ln2\) and \(\displaystyle y=\frac{π}{4}\).

    Answer

    \(\displaystyle 2\sqrt{2}t+\sqrt{2}π=\frac{du}{dt}\)

    Exercise \(\PageIndex{4}\)

    For the following exercises, find \(\displaystyle \frac{dy}{dx}\)using partial derivatives.

    16) \(\displaystyle sin(6x)+tan(8y)+5=0\)

    17) \(\displaystyle x^3+y^2x−3=0\)

    Answer

    \(\displaystyle \frac{dy}{dx}=−\frac{3x^2+y^2}{2xy}\)

    18) \(\displaystyle sin(x+y)+cos(x−y)=4\)

    19) \(\displaystyle x^2−2xy+y^4=4\)

    Answer

    \(\displaystyle \frac{dy}{dx}=\frac{y−x}{−x+2y^3}\)

    20) \(\displaystyle xe^y+ye^x−2x^2y=0\)

    21) \(\displaystyle x^{2/3}+y^{2/3}=a^{2/3}\)

    Answer

    \(\displaystyle \frac{dy}{dx}=−\sqrt[3]{\frac{y}{x}}\)

    22) \(\displaystyle xcos(xy)+ycosx=2\)

    23) \(\displaystyle e^{xy}+ye^y=1\)

    Answer

    \(\displaystyle \frac{dy}{dx}=−\frac{ye^{xy}}{xe^{xy}+e^y(1+y)}\)

    24) \(\displaystyle x^2y^3+cosy=0\)

    Exercise \(\PageIndex{5}\)

    25) Find \(\displaystyle \frac{dz}{dt}\) using the chain rule where \(\displaystyle z=3x^2y^3,x=t^4,\) and \(\displaystyle y=t^2\).

    Answer

    \(\displaystyle \frac{dz}{dt}=42t^{13}\)

    26) Let \(\displaystyle z=3cosx−sin(xy),x=\frac{1}{t},\) and \(\displaystyle y=3t.\) Find \(\displaystyle \frac{dz}{dt}\).

    27) Let \(\displaystyle z=e^{1−xy},x=t^{1/3},\) and \(\displaystyle y=t^3\). Find \(\displaystyle \frac{dz}{dt}\).

    Answer

    \(\displaystyle \frac{dz}{dt}=−\frac{10}{3}t^{7/3}×e^{1−t^{10/3}}\)

    28) Find \(\displaystyle \frac{dz}{dt}\) by the chain rule where \(\displaystyle z=cosh^2(xy),x=\frac{1}{2}t,\) and \(\displaystyle y=e^t\).

    29) Let \(\displaystyle z=\frac{x}{y},x=2cosu,\) and \(\displaystyle y=3sinv.\) Find \(\displaystyle \frac{∂z}{∂u}\) and \(\displaystyle \frac{∂z}{∂v}\).

    Answer

    \(\displaystyle \frac{∂z}{∂u}=\frac{−2sinu3}{sinv}\) and \(\displaystyle \frac{∂z}{∂v}=\frac{−2cosucosv3}{sin^2v}\)

    30) Let \(\displaystyle z=e^{x^2y}\), where \(\displaystyle x=\sqrt{uv}\) and \(\displaystyle y=\frac{1}{v}\). Find \(\displaystyle \frac{∂z}{∂u}\) and \(\displaystyle \frac{∂z}{∂v}\).

    31) If \(\displaystyle z=xye^{x/y}, x=rcosθ,\) and \(\displaystyle y=rsinθ\), find \(\displaystyle \frac{∂z}{∂r}\) and \(\displaystyle \frac{∂z}{∂θ}\) when \(\displaystyle r=2\) and \(\displaystyle θ=\frac{π}{6}\).

    Answer

    \(\displaystyle \frac{∂z}{∂r}=\sqrt{3}e^{\sqrt{3}}, \frac{∂z}{∂θ}=(2−4\sqrt{3})e^{\sqrt{3}}\)

    32) Find \(\displaystyle \frac{∂w}{∂s}\) if \(\displaystyle w=4x+y^2+z^3,x=e^{rs^2},y=ln(\frac{r+s}{t}),\) and \(\displaystyle z=rst^2\).

    33) If \(\displaystyle w=sin(xyz),x=1−3t,y=e^{1−t},\) and \(\displaystyle z=4t\), find \(\displaystyle \frac{∂w}{∂t}\).

    Answer

    \(\displaystyle \frac{∂w}{∂t}=cos(xyz)×yz×(−3)−cos(xyz)xze^{1−t}+cos(xyz)xy×4\)

    Exercise \(\PageIndex{6}\)

    For the following exercises, use this information: A function \(\displaystyle f(x,y)\) is said to be homogeneous of degree \(\displaystyle n\) if \(\displaystyle f(tx,ty)=t^nf(x,y)\). For all homogeneous functions of degree \(\displaystyle n\), the following equation is true: \(\displaystyle x\frac{∂f}{∂x}+y\frac{∂f}{∂y}=nf(x,y)\). Show that the given function is homogeneous and verify that \(\displaystyle x\frac{∂f}{∂x}+y\frac{∂f}{∂y}=nf(x,y)\).

    34) \(\displaystyle f(x,y)=3x^2+y^2\)

    35) \(\displaystyle f(x,y)=\sqrt{x^2+y^2}\)

    Answer

    \(\displaystyle f(tx,ty)=\sqrt{t^2x^2+t^2y^2}=t^1f(x,y), \frac{∂f}{∂y}=x\frac{1}{2}(x^2+y^2)^{−1/2}×2x+y\frac{1}{2}(x^2+y^2)^{−1/2}×2y=1f(x,y)\)

    36) \(\displaystyle f(x,y)=x^2y−2y^3\)

    Exercise \(\PageIndex{7}\)

    37) The volume of a right circular cylinder is given by \(\displaystyle V(x,y)=πx^2y,\) where \(\displaystyle x\) is the radius of the cylinder and \(\displaystyle y\) is the cylinder height. Suppose \(\displaystyle x\) and \(\displaystyle y\) are functions of \(\displaystyle t\) given by \(\displaystyle x=\frac{1}{2}t\) and \(\displaystyle y=\frac{1}{3}t\) so that \(\displaystyle x\) and \(\displaystyle y\) are both increasing with time. How fast is the volume increasing when \(\displaystyle x=2\) and \(\displaystyle y=5\)?

    Answer

    \(\displaystyle \frac{34π}{3}\)

    38) The pressure \(\displaystyle P\) of a gas is related to the volume and temperature by the formula \(\displaystyle PV=kT\), where temperature is expressed in kelvins. Express the pressure of the gas as a function of both \(\displaystyle V\) and \(\displaystyle T\). Find \(\displaystyle \frac{dP}{dt}\) when \(\displaystyle k=1, \frac{dV}{dt}=2\) cm3/min, \(\displaystyle \frac{dT}{dt}=12\) K/min, \(\displaystyle V=20 cm^3\), and \(\displaystyle T=20°F\).

    39) The radius of a right circular cone is increasing at \(\displaystyle 3\) cm/min whereas the height of the cone is decreasing at \(\displaystyle 2\) cm/min. Find the rate of change of the volume of the cone when the radius is \(\displaystyle 13\) cm and the height is \(\displaystyle 18\) cm.

    Answer

    \(\displaystyle \frac{dV}{dt}=\frac{1066π}{3}cm^3/min\)

    40) The volume of a frustum of a cone is given by the formula \(\displaystyle V=\frac{1}{3}πz(x^2+y^2+xy),\) where \(\displaystyle x\) is the radius of the smaller circle, \(\displaystyle y\) is the radius of the larger circle, and \(\displaystyle z\) is the height of the frustum (see figure). Find the rate of change of the volume of this frustum when \(\displaystyle x=10in.,y=12in.,\) and \(\displaystyle z=18in.\)

    A conical frustum (that is, a cone with the pointy end cut off) with height x, larger radius y, and smaller radius x.

    41) A closed box is in the shape of a rectangular solid with dimensions \(\displaystyle x,y,\) and \(\displaystyle z\). (Dimensions are in inches.) Suppose each dimension is changing at the rate of \(\displaystyle 0.5\) in./min. Find the rate of change of the total surface area of the box when \(\displaystyle x=2in.,y=3in.,\) and \(\displaystyle z=1in.\)

    Answer

    \(\displaystyle \frac{dA}{dt}=12in.^2/min\)

    42) The total resistance in a circuit that has three individual resistances represented by \(\displaystyle x,y,\) and \(\displaystyle z\) is given by the formula \(\displaystyle R(x,y,z)=\frac{xyz}{yz+xz+xy}\). Suppose at a given time the \(\displaystyle x\) resistance is \(\displaystyle 100Ω\), the \(\displaystyle y\) resistance is \(\displaystyle 200Ω,\) and the \(\displaystyle z\) resistance is \(\displaystyle 300Ω.\) Also, suppose the \(\displaystyle x\) resistance is changing at a rate of \(\displaystyle 2Ω/min,\) the \(\displaystyle y\) resistance is changing at the rate of \(\displaystyle 1Ω/min\), and the \(\displaystyle z\) resistance has no change. Find the rate of change of the total resistance in this circuit at this time.

    43) The temperature \(\displaystyle T\) at a point \(\displaystyle (x,y)\) is \(\displaystyle T(x,y)\) and is measured using the Celsius scale. A fly crawls so that its position after \(\displaystyle t\) seconds is given by \(\displaystyle x=\sqrt{1+t}\) and \(\displaystyle y=2+\frac{1}{3}t\), where \(\displaystyle x\) and \(\displaystyle y\) are measured in centimeters. The temperature function satisfies \(\displaystyle T_x(2,3)=4\) and \(\displaystyle T_y(2,3)=3\). How fast is the temperature increasing on the fly’s path after \(\displaystyle 3\) sec?

    Answer

    \(\displaystyle 2°C/sec\)

    44) The \(\displaystyle x\) and \(\displaystyle y\) components of a fluid moving in two dimensions are given by the following functions: \(\displaystyle u(x,y)=2y\) and \(\displaystyle v(x,y)=−2x; x≥0;y≥0.\) The speed of the fluid at the point \(\displaystyle (x,y)\) is \(\displaystyle s(x,y)=\sqrt{u(x,y)^2+v(x,y)^2}\). Find \(\displaystyle \frac{∂s}{∂x}\) and \(\displaystyle \frac{∂s}{∂y}\) using the chain rule.

    45) Let \(\displaystyle u=u(x,y,z),\) where \(\displaystyle x=x(w,t),y=y(w,t),z=z(w,t),w=w(r,s),\) and \(\displaystyle t=t(r,s).\) Use a tree diagram and the chain rule to find an expression for \(\displaystyle \frac{∂u}{∂r}\).

    Answer

    \(\displaystyle \frac{∂u}{∂r}=\frac{∂u}{∂x}(\frac{∂x}{∂w}\frac{∂w}{∂r}+\frac{∂x}{∂t}\frac{∂t}{∂r})+\frac{∂u}{∂y}(\frac{∂y}{∂w}\frac{∂w}{∂r}+\frac{∂y}{∂t}\frac{∂t}{∂r})+\frac{∂u}{∂z}(\frac{∂z}{∂w}\frac{∂w}{∂r}+\frac{∂z}{∂t}\frac{∂t}{∂r})\)


    6.4E: Exercises is shared under a not declared license and was authored, remixed, and/or curated by LibreTexts.

    • Was this article helpful?